Question

CAPM For a risky return r, CAPM equation is Er -r- B(E[rm] -r), where r is risk-free rate, Tm is market return, and is loadin
0 0
Add a comment Improve this question Transcribed image text
Answer #1

8. Inconsistent - With the increase in beta , the expected return should rise while it is decreasing in this case.

9.Inconsistent - With the same variance, the return on bond is fluctuating

10. Consistent - The expected return and variance are moving in the same direction making it consistent.

11.Consistent - The beta and expected return are moving in the same direction, making it consistent.

12. Consistent - The beta and market return are moving in the same direction making it consistent, where it means if beta increases , return increases and vice versa

Add a comment
Know the answer?
Add Answer to:
CAPM For a risky return r, CAPM equation is Er -r- B(E[rm] -r), where r is risk-free rate, Tm is ...
Your Answer:

Post as a guest

Your Name:

What's your source?

Earn Coins

Coins can be redeemed for fabulous gifts.

Not the answer you're looking for? Ask your own homework help question. Our experts will answer your question WITHIN MINUTES for Free.
Similar Homework Help Questions
  • Suppose that CAPM holds. Let Rf  denote the risk free rate, E(RM) the expected return of the...

    Suppose that CAPM holds. Let Rf  denote the risk free rate, E(RM) the expected return of the market portfolio, and sigmaMthe standard deviation of the market portfolio. Now consider some portfolio on the capital market line, with expected return E(R) and standard deviation sigma. What is the beta of this portfolio? Select one: 1. E(R)/sigma 2. sigmaM/sigma 3. sigma/sigmaM 4. E(RM)-Rf

  • a) If the CAPM is correct, what would be the expected return of a risky asset...

    a) If the CAPM is correct, what would be the expected return of a risky asset with a beta of 1.2, given a risk free rate of 3% and an expected market risk premium of 4.5%? b) If the CAPM is correct, what would be the expected return of a risky asset with a beta of 0.8, given a risk free rate of 4% and an expected return of the market of 9%

  • CAPM data: Market portfolio: Risk-free asset: Om = 0.2 E[RM]=18%, R, = 6% T-bills are also...

    CAPM data: Market portfolio: Risk-free asset: Om = 0.2 E[RM]=18%, R, = 6% T-bills are also available. They are considered riskless and have a corresponding rate of return. You have $20,000 to invest. a) What are Br-Bills, and 07-Bills? (1 mark) b) Consider Portfolio X comprised of T-Bills and a $25,000 investment in the market portfolio i) Find 0,- (1 mark) ii) Solve for Br. (1 marks) c) Determine the weights of T-Bills and the market portfolio that combined would...

  • 3. Expected return and CAPM Suppose the risk-free rate is 4% and the market portfolio and...

    3. Expected return and CAPM Suppose the risk-free rate is 4% and the market portfolio and stock j have the following return distributions: Probability in tot Market return -.15 .05 .15 .20 Return for i --30 .00 .20 .50 a. Find the expected market return, Im. b. Find the variance of the market return, c. Find the expected return for stock j, r;. d. Find the covariance of j and the market, Oim. e. What is J's beta?

  • 3. Expected return and CAPM Suppose the risk-free rate is 4% and the market portfolio and...

    3. Expected return and CAPM Suppose the risk-free rate is 4% and the market portfolio and stock j have the following return distributions: Probability in tot Market return -.15 .05 .15 .20 Return for i --30 .00 .20 .50 a. Find the expected market return, Im. b. Find the variance of the market return, c. Find the expected return for stock j, r;. d. Find the covariance of j and the market, Oim. e. What is J's beta?

  • Er (%) 0.2 Consider the CAPM framework. Suppose that you currently have 40% of your wealth...

    Er (%) 0.2 Consider the CAPM framework. Suppose that you currently have 40% of your wealth in Treasury Bills, risk-free, and 60% in the four assets below Asset i Bi Wi i = 1 8.5 0.2 0.1 i = 2 13.1 0.8 0.1 i = 3 16.6 1.2 0.2 i = 4 18.7 1.4 Let the four assets be traded in a market M with Erm = 15% and let the risk-free rate be rf = 4%, answer the following...

  • Consider the two (excess return) index-model regression results for stocks A and B. The risk-free rate...

    Consider the two (excess return) index-model regression results for stocks A and B. The risk-free rate over the period was 7%, and the market's average return was 13%. Performance is measured using an index model regression on excess returns. Index model regression estimates R-square Residual standard deviation, o(e) Standard deviation of excess returns Stock A 1% + 1.2 (rm -rf) 0.629 11.2% 22.5% Stock B 2% + 0.8(rm -rf) 0.463 20% 26.7% a. Calculate the following statistics for each stock:...

  • (2*5) Consider a market with many risky assets and a risk-free security. Asset’s returns are not...

    (2*5) Consider a market with many risky assets and a risk-free security. Asset’s returns are not perfectly correlated. All the CAPM assumptions hold and the market is in equilibrium. The risk-free rate is 5%, the expected return on the market is 15%. Mr. T and Mrs. R are two investors with mean-variance utility functions and different risk-aversion coefficients. They both invest into efficient portfolios composed of the market portfolio and the risk-free security. Mr. T’s portfolio has an expected return...

  • Suppose there are two assets, one is risk-free and one is risky. The risk-free asset has a sure r...

    Suppose there are two assets, one is risk-free and one is risky. The risk-free asset has a sure rate of return rj, the risky asset has a random rate of return r. Suppose the utility function of an investor is U(x) =--. The initial wealth is wo, the dollar amount invested in the risky asset is θ. r is normally distributed with mean μ and variance σ2. Based on the maximum utility framework, find the optimal investment strategy 6. (25...

  • The risk-free rate is 4.5%, the market risk premium = ( E(Rm) - Rf) is 10.1%,...

    The risk-free rate is 4.5%, the market risk premium = ( E(Rm) - Rf) is 10.1%, and the stock’s beta is 1.3. What is the required rate of return on the stock, E(Ri)? Use the CAPM equation.

ADVERTISEMENT
Free Homework Help App
Download From Google Play
Scan Your Homework
to Get Instant Free Answers
Need Online Homework Help?
Ask a Question
Get Answers For Free
Most questions answered within 3 hours.
ADVERTISEMENT
ADVERTISEMENT
Active Questions
ADVERTISEMENT